Difference between revisions of "2021 Fall AMC 10A Problems/Problem 5"

(Solution 1)
(Tag: New redirect)
 
(One intermediate revision by the same user not shown)
Line 1: Line 1:
The six-digit number <math>\underline{2}\,\underline{0}\,\underline{2}\,\underline{1}\,\underline{0}\,\underline{A}</math> is prime for only one digit <math>A.</math> What is <math>A?</math>
+
#REDIRECT [[2021_Fall_AMC_12A_Problems/Problem_4]]
 
 
<math>(\textbf{A})\: 1\qquad(\textbf{B}) \: 3\qquad(\textbf{C}) \: 5 \qquad(\textbf{D}) \: 7\qquad(\textbf{E}) \: 9</math>
 
 
 
== Solution 1==
 
 
 
By divisibility rules, when <math>A=1,</math> the number <math>202101</math> is divisible by <math>3.</math> When <math>A=3,</math> the number <math>202103</math> is divisible by <math>11.</math> When <math>A=5,</math> the number <math>202105</math> is divisible by <math>5.</math> When <math>A=7,</math> the number <math>202107</math> is divisible by <math>3.</math> Thus, by the process of elimination we have that the answer is <math>\boxed{\textbf{(E)}}</math>
 
 
 
~NH14
 
 
 
==See Also==
 
{{AMC10 box|year=2021 Fall|ab=A|num-b=4|num-a=6}}
 
{{MAA Notice}}
 

Latest revision as of 19:12, 23 November 2021